Maternity EAQ Study Guide

Lakukan tugas rumah & ujian kamu dengan baik sekarang menggunakan Quizwiz!

Which is an appropriate response to a client who is 8 weeks pregnant and is concerned because she doesn't 'feel like making love to her husband since becoming pregnant' and she doesn't 'think he understands'? 'Was this an issue you'd experienced before your pregnancy?' 'Why don't you feel like making love with your husband anymore?' 'A decrease in libido is common during the first trimester of pregnancy.' 'I'm sure your husband will understand that this feeling is related to your pregnancy.'

'A decrease in libido is common during the first trimester of pregnancy.' Often the pregnant woman experiences a decrease in sexual desire during the first trimester, probably as a result of nausea and vomiting; if couples are informed about this, they are less likely to become distressed. The client has already stated that the problem began with pregnancy. The client is asking the nurse for information; the client may be unable to tell the nurse. Stating that the client's husband will understand that this feeling is related to the client's pregnancy does not tell the client why this feeling is occurring; furthermore, it offers false reassurance.

Which direction regarding sleeping position would the nurse give to a client who is 8 months' pregnant? ' Try to sleep on your stomach.' 'Turn from side to side when in bed.' 'Elevate the head of the bed on blocks.' 'Place pillows under your knees for sleep.'

'Turn from side to side when in bed.' The side-lying position will relieve back pressure; it also promotes uterine perfusion and fetal oxygenation. At 32 weeks' gestation the abdomen is too distended for the pregnant woman to lie in the prone position. Elevating the head of the bed will not relieve back pressure; it is used to limit gastroesophageal reflux. Lying on the back is contraindicated because it puts pressure on the vena cava, resulting in hypotension and uteroplacental insufficiency. Pillows under the knees are contraindicated because they place pressure on the popliteal area, which compresses the venous circulation, increasing the risk of thrombophlebitis.

Where is the presenting part of the fetus when station is -1? 1 cm above the ischial spines 1 cm below the ischial spines Visible at the vaginal opening At the level of the ischial spines

1 cm above the ischial spines Station -1 signifies that the fetal head is 1 cm above the ischial spines and has not reached the vaginal canal. When the fetal head is 1 cm below the ischial spines, it is at station +1. When the fetal head is visible at the vaginal opening, it is at station +4. When the fetal head is level with the ischial spines, it is at station 0.

Which lecithin/sphingomyelin (L/S) ratio is indicative of fetal lung maturity? 1:1 1.4:1 1.8:1 2:1

2:1 The detection of the presence of pulmonary surfactants, surface-active phospholipids, in amniotic fluid has been used to determine the degree of fetal lung maturity, or the ability of the lungs to function after birth. Lecithin (L) is the most critical alveolar surfactant required for postnatal lung expansion. It is detectable at approximately 21 weeks and increases after week 24. Another pulmonary phospholipid, sphingomyelin (S), remains constant in amount. The measure of lecithin in relation to sphingomyelin, or the L/S ratio, is used to determine fetal lung maturity. When the L/S ratio reaches 2:1, the fetus' lungs are considered mature. The ratios of 1:1, 1.4:1, and 1.8:1 are incorrect.

Which percentage of all cervical cancers occurs during pregnancy? 3% 6% 9% 12%

3% Careful assessment of all pregnant women is important because approximately 3% of all invasive cervical cancers occur during pregnancy. The other answer options (6%, 9%, and 12%) are all too high for the percentage of all cervical cancers that occur during pregnancy.

Which pH value of amniotic fluid is indicated by a Nitrazine test strip that turns deep blue? 4.5 5.5 6.5 7.5

7.5 Amniotic fluid changes the color of a nitrazine strip from yellow to deep blue if the pH of the fluid is 7.5. A pH of 4.5, 5.5, or 6.5 would result in a test strip of yellow, olive yellow, or blue green, respectively.

At 39 weeks' gestation a client asks the nurse about the difference between true and false labor. Which information regarding true labor contractions would the nurse include in a response to the client's question? Usually fluctuate in length Continuous, without relaxation Related to time of membrane rupture Accompanied by progressive cervical dilation

Accompanied by progressive cervical dilation Progressive cervical dilation is the only positive sign of true labor; the cervix dilates in response to regular, coordinated uterine contractions. The contractions of true labor increase in length and intensity. A continuous contraction may have an adverse effect on the fetus; immediate intervention is required. The membranes may rupture before contractions begin; more frequently they rupture after true labor is established.

Which phrase would the nurse use to document a fetal heart rate (FHR) increase of 15 beats over the baseline rate of 135 beats per minute that lasts 15 seconds? An acceleration An early increase A sonographic motion A tachycardic heart rate

An acceleration An acceleration is an abrupt increase in FHR above the baseline of 15 beats/min for 15 seconds; if the acceleration persists for more than 10 minutes, it is considered a change in baseline rate. Early decelerations, not increases, occur. An early deceleration starts before the peak of the uterine contraction and returns to baseline when the uterine contraction ends. A sonographic motion is not a term used in fetal monitoring. A tachycardic FHR is one faster than 160 beats per minute.

Which physiological alteration does the nurse expect in a client's hematological system during the second trimester of pregnancy? An increase in hematocrit An increase in blood volume A decrease in sedimentation rate A decrease in white blood cells (WBCs)

An increase in blood volume The blood volume increases by approximately 50% during pregnancy. Peak blood volume occurs between 30 and 34 weeks' gestation. The hematocrit decreases as a result of hemodilution. The sedimentation rate increases because of a decrease in plasma proteins. WBCs count increases somewhat starting in the second trimester and peak in the third. WBCs are in the 5000 to 15,000 range during pregnancy.

Which is the immediate nursing action when a client's membranes rupture spontaneously, releasing clear, odorless fluid? Change the bedding. Notify the practitioner. Assess the fetal heart rate (FHR). Obtain the client's blood pressure.

Ans: Assess the fetal heart rate (FHR). The FHR will reflect how the fetus tolerated the rupture of the membranes; if there is compression of the cord, it will be reflected in a change in the FHR. Although the client's comfort is important, addressing comfort by changing the bedding is not the priority. Although the practitioner should be notified, it is not the priority. Blood pressure is not influenced by rupture of the membranes.

Which information about adolescent growth and development would the nurse need to understand before discussing changes in body size to a 16-year-old adolescent at 24 weeks' gestation? Adolescents generally regain their figures 2 weeks after the birth, so size is of moderate concern. Adolescents are in a high-risk category, so weight gain should be limited to prevent complications. Body image is very important to adolescents; therefore, pregnant teenagers are overly concerned about body size. Physiological growth in adolescents is more rapid than in adults, so the gravid size is larger than that of an adult woman.

Ans: Body image is very important to adolescents; therefore, pregnant teenagers are overly concerned about body size. Because of the changes in body size, the pregnant adolescent may feel insecure as she struggles to establish her identity. There are no data to support the statement that adolescents generally regain their figures 2 weeks after the birth. The optimal weight gain for an adolescent is at the upper range for her body mass index; this will help prevent complications, so limiting weight gain does not prevent complications. Although physiological growth is rapid, the adolescent's gravid size falls within the expected parameters for pregnant women and is not larger than that of adult women.

A prenatal client's vaginal mucosa is noted to have a purplish discoloration. Which sign would be documented in the client's clinical record? Hegar Goodell Chadwick Braxton-Hicks

Ans: Chadwick A purplish coloration, called the Chadwick sign, results from the increased vascularity and blood vessel engorgement of the vagina. The Hegar sign is softening of the lower uterine segment. The Goodell sign is softening of the cervix. After the fourth month of pregnancy, irregular, painless uterine contractions, called Braxton-Hicks contractions, can be felt through the abdominal wall.

The nurse suspects a thrombus after assessing a client who has pain in her right calf 2 days after a cesarean birth. Which is the nurse's immediate action? Confine client to bed. Apply warm soaks. Perform leg exercises. Massage the affected area.

Ans: Confine client to bed. When a thrombus is suspected but before a definitive diagnosis is made, the client should be confined to bed so that further complications may be avoided. Applying warm soaks may cause vasodilation, which could allow a thrombus to dislodge and circulate freely. If a thrombus is present, massage may dislodge it and lead to a pulmonary embolism.

A client who is at 20 weeks' gestation visits the prenatal clinic for the first time. Assessment reveals temperature of 98.8°F (37.1°C), pulse of 80 beats per minute, blood pressure of 128/80 mm Hg, weight of 142 lb (64.4 kg) (prepregnancy weight was 132 lb [59.9 kg]), fetal heart rate (FHR) of 140 beats per minute, urine that is negative for protein, and fasting blood glucose level of 92 mg/dL (5.2 mmol/L). Which would the nurse do after making these assessments? Report the findings because the client needs immediate intervention. Document the results because they are expected at 20 weeks' gestation. Record the findings in the medical record because they are not within the norm but are not critical. Prepare the client for an emergency admission because these findings may represent jeopardy to the client and fetus.

Ans: Document the results because they are expected at 20 weeks' gestation. All data presented are expected for a client at 20 weeks' gestation and should be documented. There is no need for immediate intervention or an emergency admission because all findings are expected.

Which technique would the nurse suggest to a laboring woman's partner that involves gently stroking the woman's abdomen in rhythm with her breathing during a contraction? Massage Effleurage Acupressure Counterpressure

Ans: Effleurage Effleurage is the gentle stroking of the abdomen in rhythm with her breathing during a contraction. Massage is the application of therapeutic touch and pressure on the body. Acupressure is the application of pressure along special acupressure points. Counterpressure is the application of pressure to the sacrum during a contraction.

Which direction would the nurse give a client in preparation for ultrasonography at the end of her first trimester? Empty her bladder. Avoid eating for 8 hours. Take a laxative the night before the test. Increase fluid intake for 1 hour before the procedure.

Ans: Increase fluid intake for 1 hour before the procedure. In the first trimester when fluid fills the bladder, the uterus is pushed up toward the abdominal cavity for optimum ultrasound viewing. The bladder must be full, not empty, for better visualization of the uterus. The gastrointestinal tract is not involved in ultrasound preparation, so directing the client to not eat for 8 hours before the test or to take a laxative would not be appropriate.

Which common problem affects the client in labor when an external fetal monitor has been applied to her abdomen? Intrusion on movement Inability to take sedatives Interference with breathing techniques Increased frequency of vaginal examinations

Ans: Intrusion on movement Because the client is attached to a machine and movement may alter the tracings, movement is discouraged. Placement of the external monitor leads does not interfere with the administration of sedatives. An external monitor does not interfere with breathing techniques. An external monitor does not necessitate more frequent vaginal examinations.

Which condition is detected by an alpha-fetoprotein test? Kidney defects Cardiac anomalies Neural tube defects Urinary tract anomalies

Ans: Neural tube defects The alpha-fetoprotein test detects neural tube defects, Down syndrome, and other congenital anomalies. It is a screening test that affords a tentative diagnosis; confirmation requires more definitive testing. Anomalies of the kidneys, heart, and urinary tract are not revealed by the alpha-fetoprotein test.

Which high-risk nutritional practice must be assessed for when a pregnant client is found to be anemic? Pica Caffeine intake Alcohol abuse Artificial sweetener use

Ans: Pica The practice of pica, especially the ingestion of heavy metals, must be considered when pregnant women are found to be anemic. Caffeine, alcohol, and artificial sweeteners are not directly linked to anemia in pregnant women.

Which is a sensory simulation strategy a laboring client can use as a nonpharmacological strategy for pain management? Gentle massage of the abdomen Biofeedback-assisted relaxation techniques Application of a heat pack to the lower back Selecting a focal point and beginning breathing technique

Ans: Selecting a focal point and beginning breathing technique Use of a focal point and breathing techniques are sensory simulation strategies. Heat and massage are cutaneous stimulation strategies; biofeedback-assisted relaxation is a cognitive strategy.

When a client at 39 weeks' gestation arrives at the birthing suite she says, 'I've been having contractions for 3 hours, and I think my water broke.' Which action would the nurse take to confirm that the membranes have ruptured? Take the client's oral temperature. Test the leaking fluid with nitrazine paper. Obtain a clean-catch urine specimen. Inspect the perineum for leaking fluid.

Ans: Test the leaking fluid with nitrazine paper. Nitrazine paper will turn dark blue if amniotic fluid is present; it remains the same color in the presence of urine. Temperature assessment is not specific to ruptured membranes at this time; vital signs are part of the initial assessment. Although this may be done as part of the initial assessment, a urine test is unrelated to leakage of amniotic fluid. Inspecting the perineum for leaking fluid will not confirm rupture of the membranes.

A nonstress test (NST) is scheduled for a client with mild preeclampsia. During an NST, the client asks what it means when the fetal heart rate goes up every time the fetus moves. Which is an appropriate response? 'These accelerations are a sign of fetal well-being.' 'These accelerations indicate fetal head compression.' 'Umbilical cord compression is causing these accelerations.' 'Uteroplacental insufficiency is causing these accelerations.'

Ans: These accelerations are a sign of fetal well-being.' The NST is performed before labor begins. Accelerations with movement and a baseline variability of 5 to 15 beats/min indicate fetal well-being. This reactive NST is considered positive. Early decelerations are associated with fetal head compression during a contraction stress test (CST) or during labor. Variable decelerations are associated with cord compression during a CST or during labor. Late decelerations during a CST or during labor are associated with uteroplacental insufficiency.

Morning sickness generally disappears by the end of which month? Fifth month Third month Fourth month Second month

Ans: Third month Because of a decrease in chorionic gonadotropin, morning sickness seldom persists beyond the first trimester. Morning sickness usually ends at the end of the third month, not the second month, when the chorionic gonadotropin level falls. It is still present in the second month because of the high level of chorionic gonadotropin but has usually diminished by the fifth month.

In which location is the presenting part of the fetus when it is at 0 station? Entering the vagina Floating within the bony pelvis At the level of the ischial spines Above the level of the ischial spines

At the level of the ischial spines The ischial spines are used as landmarks in relation to the fetus's head because they reflect the progression of labor; 0 station indicates that the presenting part is at the ischial spines. When the head enters the vagina, it is below the ischial spines and its position is designated with positive numbers (+1 to +4). When the presenting part is floating, the fetus is at -5 station. A position above the ischial spines is designated by a minus number (-1 to -4).

In which area of the maternal pelvis is the presenting part when station is +1? Above the pelvic inlet Below the ischial spines Entering the pelvic inlet Visible at the vaginal opening

Below the ischial spines A station of +1 indicates that the presenting part is 1 cm below the ischial spines. The presenting part is now past the points of engagement, the ischial spines. When the presenting part is entering the pelvic inlet, it is said to be at 0 station. The presenting part must be at +3 to +4 station to be visible at the vaginal opening.

In which location would the Doppler ultrasound transducer be placed to best auscultate fetal heart tones when the fetus is in the right occiput posterior (ROP) position? Above the umbilicus in the midline Above the umbilicus on the left side Below the umbilicus on the right side Below the umbilicus near the left groin

Below the umbilicus on the right side Fetal heart tones are best auscultated through the fetal back. In this case the presenting part is in the ROP position; the back is below the umbilicus and on the right side. Above the umbilicus in the midline is the placement that should be used when the fetus is lying in the midline in a breech position. Placement above the umbilicus on the left side is appropriate when the fetus is in the left sacrum anterior position. Placement below the umbilicus near the left groin is appropriate when the fetus is in the left occiput anterior or left occiput posterior position.

Which prenatal test provides the earliest diagnosis of fetal defects? Nonstress test Amniocentesis Chorionic villus sampling Percutaneous umbilical blood sampling

Chorionic villus sampling Chorionic villus sampling may be performed between 10 and 12 weeks' gestation. The nonstress test, which is not invasive, is a technique used for antepartum evaluation of the fetus; it does not reveal fetal defects. Amniocentesis may be performed after 14 weeks' gestation, when sufficient amniotic fluid is available. Direct access to the fetal circulation with percutaneous umbilical blood sampling may be performed during the second and third trimesters.

Which would the nurse ask the postpartum client to do before assessing her uterine fundus? Drink fluids. Empty her bladder. Perform the Valsalva maneuver. Assume the semi-Fowler position.

Empty her bladder. Having the client empty her bladder will help ensure accurate assessment of fundal height. A full bladder may promote a boggy uterus and may elevate the uterus upward and toward the client's right side. There is no need to drink fluids before this assessment; however, the client should drink at least 2 L of fluid a day during the postpartum period. The Valsalva maneuver has no effect on the assessment of fundal height. Assessing the fundus while the client is in the semi-Fowler position will result in an inaccurate assessment. The bed should be flat, and the client should assume the supine position.

Which direction would the nurse give to a 3-day postpartum client who reports that her breasts are so painful that she dreads breast-feeding the baby? Take an analgesic before breast-feeding. Formula-feed the baby for 2 days or until the pain is relieved. Express some milk manually before feeding to relieve pressure. Limit fluids for 24 hours to allow the breasts to adjust to filling with milk.

Express some milk manually before feeding to relieve pressure. The pressure and tenderness resulting from accumulated milk can be relieved by manually expressing some of the fluid before feeding. Pain medication may be taken if other measures are unsuccessful; however, medication can be transferred to the infant through breast milk. Also, giving medication is a dependent function of the nurse that requires a prescription. Breast-feeding, not formula feeding, should continue as a means of limiting engorgement and aiding milk production. The mother should not limit fluids, especially if she is breast-feeding.

During the assessment of a client in labor, the cervix is determined to be dilated 4 cm. Which stage of labor would the nurse record? First Second Prodromal Transitional

First The first stage of labor is from zero cervical dilation to full cervical dilation (10 cm). The second stage is from full cervical dilation to delivery. The prodromal stage is before cervical dilation begins. The transitional phase is the first stage of labor, from 8 cm of dilation to 10 cm of dilation.

Which prenatal teaching is most applicable for a client who is between 13 and 24 weeks' gestation? Infant care, travel to the hospital, and signs of labor Growth of the fetus, body changes, and nutritional guidance Interventions for nausea and vomiting, urinary frequency, and anticipated care Danger signs of preeclampsia, relaxation breathing techniques, and signs of labor

Growth of the fetus, body changes, and nutritional guidance Awareness of the fetus as an individual and the expected changes of pregnancy lead the client to seek information regarding fetal growth, body changes, and nutrition. Information on infant care, travel to the hospital, signs of labor, signs of preeclampsia, and relaxation breathing techniques are appropriate in the last trimester. Interventions for nausea and vomiting, urinary frequency, and anticipated care are appropriate for the first trimester.

In the second hour after the client gives birth, her uterus is firm, above the level of the umbilicus, and to the right of midline. Which nursing action is an appropriate response to this situation? Having the client empty her bladder Watching for signs of retained secundines Massaging the uterus vigorously to prevent hemorrhage Explaining to the client that this is a sign of uterine stabilization

Having the client empty her bladder A full bladder elevates the uterus and displaces it to the right. Even though the uterus feels firm, it may relax enough to foster bleeding. The bladder should be emptied to improve uterine tone. Watching for signs of retained secundines may be done if emptying the bladder does not rectify the situation. If parts of the placenta, umbilical cord, or fetal membranes are not fully expelled during the third stage of labor, their retention limits uterine contraction and involution; a boggy uterus and bleeding may be evident. Vigorous massage tires the uterus, and even with massage the uterus is unable to contract over a full bladder. Explaining to the client that this is a sign of uterine stabilization is not accurate; the uterus will not remain contracted over a full bladder.

Which would the nurse assess for in pregnant women who present with signs of physical abuse or neglect? Alcohol abuse Substance use Human trafficking Occupational activities

Human trafficking Nurses would assess for human trafficking in women who present with signs of physical abuse or neglect, such as scarring, bruises, burns, bald patches, or tattoos. Women who abuse alcohol or use substances do not usually present with signs of physical abuse or neglect. Occupational activities do not result in signs of physical abuse or neglect.

A client who has had a cesarean birth is being discharged. Which statement indicates to the nurse that further teaching is required? 'I may take an oxycodone/acetaminophen (Percocet) tablet if my incision hurts.' 'I should take a mild laxative if I don't have a bowel movement.'' I can start mild exercises once my incision has stopped hurting.'' I don't need perineal care because I didn't give birth through the vagina.'

I don't need perineal care because I didn't give birth through the vagina.' After a cesarean birth, the client has the same vaginal discharge (lochia) as a client who gave birth vaginally. Perineal care is necessary to prevent an ascending infection. Mild laxatives are permitted if needed. Oxycodone/acetaminophen (Percocet) or a similar analgesic usually is prescribed. Mild exercise is not contraindicated if there is no incisional pain.

While conducting prenatal teaching, the nurse explains to clients that there is an increase in vaginal secretions during pregnancy called leukorrhea. Which factor does the nurse identify as the cause of this increase? Decreased metabolic rate Increased production of estrogen Secretion from the Bartholin glands Supply of sodium chloride to the vaginal cells

Increased production of estrogen Increased estrogen production during pregnancy causes hyperplasia of the vaginal mucosa, which leads to increased production of mucus by the endocervical glands. The mucus contains exfoliated epithelial cells. Increased (not decreased) metabolism leads to systemic changes but does not increase vaginal discharge. The amount of secretion from the Bartholin glands, which lubricates the vagina during intercourse, remains unchanged during pregnancy. There is no additional supply of sodium chloride to the vaginal cells during pregnancy.

Which action would the nurse take on finding that a client receiving oxytocin intravenously has a soft, boggy fundus a half-hour after a cesarean birth? Elevate the client's legs. Massage the client's fundus. Increase the client's oxytocin drip rate. Examine the client's perineum for bleeding.

Massage the client's fundus. Gentle massage stimulates muscle fibers, resulting in firming the tone of the fundus; it also helps expel any clots that may be interfering with contraction of the fundus. Elevating the client's legs will increase return of blood from the extremities but will not improve the tone of the client's fundus. Increasing the client's oxytocin drip rate will be done if uterine massage is ineffective. Examining the client's perineum for bleeding should not be the first action at this time; gentle massage to contract the fundus is the priority.

Which is the priority nursing intervention for the postpartum client whose fundus is 3 fingerbreadths above the umbilicus, boggy, and midline? Massaging the uterine fundus Helping the client to the bathroom Assessing the peri-pad for the amount of lochia Administering intramuscular methylergonovine (Methergine) 0.2 mg

Massaging the uterine fundus A uterus that is displaced and above the umbilicus indicates relaxation of the uterine muscle. Fundal massage is necessary to stimulate uterine contractions. The status of the fundus and correction of uterine relaxation must be done before the client is helped to the bathroom, the amount of lochia is assessed, ormethylergonovine (Methergine) is administered.

An adolescent who gave birth 12 hours ago continually talks on the phone to her friends and does not respond when her new baby cries. Which is the priority intervention at this time? Initiating a social services consult Calling the psychiatric team for an intervention Arranging for the client's parent to speak with her Modeling appropriate behaviors that encourage infant bonding

Modeling appropriate behaviors that encourage infant bonding All women go through several phases of adapting to the role of mother. An adolescent may still need time to adjust to her new role, especially if she has just given birth in the past 24 hours. By modeling appropriate behavior, the nurse demonstrates appropriate maternal skills to the adolescent. This will assist her as she makes the transition into her new role as a mother. If this behavior continues and does not improve before discharge, social services may need to get involved, but a consult is not needed in this early phase. A psychiatric consult is not necessary because this is not a psychiatric illness. The adolescent's parent is an important part of the plan, especially if the adolescent is going home to her or his house, but the relationship between the two needs to be assessed to see what role he or she will play in this new parent-child relationship.

Which statement indicates a client understands the meaning of having a reactive nonstress test? Normal because of increases in fetal heart rate (FHR) with fetal movement Abnormal because of a decrease in FHR between contractions Abnormal because of variability in FHR with each contraction Normal because the FHR remained unchanged with maternal movement

Normal because of increases in fetal heart rate (FHR) with fetal movement A reactive nonstress test is an expected finding because there are 2 or more increases in FHR greater than 15 beats/min associated with fetal movement; it suggests fetal well-being. There are no uterine contractions during a nonstress test. Maternal movement has no bearing on nonstress test readings; fetal movements and FHR are monitored.

During a routine prenatal office visit at 26 weeks' gestation, a client states that she is getting fat all over and that she even needed to buy bigger shoes. Which is the next nursing action? Obtaining the client's weight and blood pressure Reassuring the client that weight gain is expected Supporting the client's decision to buy comfortable shoes Teaching the client about the importance of limiting fatty foods and sweets

Obtaining the client's weight and blood pressure The client's weight and blood pressure help the nurse determine whether an unusual weight gain or an increase in blood pressure has occurred; both of these findings are early signs of preeclampsia. The data suggest a greater-than-expected weight gain. Supporting the client's decision to buy comfortable shoes ignores the possibility that the edema and weight gain are related to preeclampsia. The weight gain may not be caused by inappropriate dietary intake, such as fatty foods and sweets, but rather by an underlying pathologic condition.

Which complication would the nurse anticipate when a client who is 36 weeks' pregnant presents with swelling of the face, blurred vision, and epigastric discomfort? Preeclampsia Placenta previa Gestational diabetes Hyperemesis gravidarum

Preeclampsia Swelling of the face, blurred vision, and epigastric discomfort are classic signs of preeclampsia. Placenta previa, gestational diabetes, and hyperemesis gravidarum do not present with swelling of the face, blurred vision, and epigastric discomfort.

On a routine prenatal visit, which is the sign or symptom that a healthy primigravida at 20 weeks' gestation will most likely report for the first time? Quickening Palpitations Pedal edema Vaginal spotting

Quickening The recognition of fetal movement or quickening commonly occurs in primigravidas at 18 to 20 weeks' gestation; it is felt about 2 weeks earlier in multigravidas. Palpitations should not occur in the healthy primigravidas. Pedal edema may occur at the end of the pregnancy as the gravid uterus presses on the femoral arteries, impeding circulation. Immediate follow-up care is required when it occurs this early in the pregnancy. Vaginal spotting is abnormal and requires immediate follow-up care.

Which fetal heart tracing during labor can most likely result in fetal hypoxia and metabolic acidosis? Accelerations Variable decelerations Fetal heart rate variability Recurrent late decelerations

Recurrent late decelerations Recurrent late decelerations can most likely result in fetal hypoxia and metabolic acidosis. Accelerations, variable decelerations, and fetal heart rate variability are all expected and will be evaluated but are not the most likely to result in fetal hypoxia and metabolic acidosis.

A client in the active phase of the first stage of labor begins to tremble, becomes very tense during contractions, and is quite irritable, saying repeatedly, 'I can't take this a minute longer.' Which is the best explanation for this behavior? There was no preparation for labor. She needs an analgesic for pain. She is entering the transition phase of labor. Hypertonic uterine contractions are developing.

She is entering the transition phase of labor. The contractions become stronger, last longer, and occur erratically during the transition phase; the intervals between contractions become shorter than the contractions themselves; the client needs to apply a great deal of concentration and effort to pace her breathing with each contraction. Even clients who have been adequately prepared will experience these behaviors during the transition phase of the first stage of labor. Administration of an analgesic at this time may reduce the effectiveness of labor and depress the fetus. There is no indication that the contractions are hypertonic.

Which is the expected color and consistency of amniotic fluid at 36 weeks' gestation? Clear, dark amber colored, and containing shreds of mucus Straw colored, clear, and containing little white specks Milky, greenish yellow, and containing shreds of mucus Greenish yellow, cloudy, and containing little white specks

Straw colored, clear, and containing little white specks By 36 weeks' gestation, amniotic fluid should be pale yellow or straw-colored with small particles of vernix caseosa present. Dark amber-colored fluid suggests the presence of bilirubin, an ominous sign. Greenish-yellow fluid may indicate the presence of meconium and suggests fetal compromise. Cloudy fluid suggests the presence of purulent material.

Which action would the nurse take when a client who is performing patterned, paced breathing during the transition phase of labor experiences tingling and numbness of the fingertips? Tell the client to breathe into a paper bag. Place an oxygen mask over the client's face. Call the primary health care provider to report the client's response. Instruct the client to begin taking slow, deep breaths.

Tell the client to breathe into a paper bag. A paper bag enables the client to rebreathe carbon dioxide, which helps correct the respiratory alkalosis resulting from hyperventilation. The client's oxygen level is increased; the client needs to increase the carbon dioxide level and decrease the oxygen level. The client should rebreathe her own exhalations first; if alkalosis persists, more intensive treatment may be needed. There is no need to place an oxygen mask over the client's face because the client already has an increased level of oxygen, nor is it necessary to call the primary health provider to report the client's response. Carbon dioxide is too dilute in room atmosphere; deep breaths will not resolve the alkalosis.

Which statement made by a pregnant client after a prenatal class on fetal growth and development indicates the need for additional teaching? ' The baby is smaller if the mother smokes.' 'The baby gets food from the amniotic fluid.' 'The baby's oxygen is provided by the mother.' 'The baby's umbilical cord has 2 arteries and 1 vein.'

The baby gets food from the amniotic fluid.' The amniotic fluid serves as a protective environment; the fetus depends on the placenta, along with the umbilical blood vessels, to supply blood containing nutrients and oxygen. 'The baby is smaller if the mother smokes,' 'The baby's oxygen is provided by the mother,' and 'The baby's umbilical cord has 2 arteries and 1 vein' are all true statements, and further teaching would not be required.

Which information would tell the nurse if a woman at 40 weeks' gestation having contractions is in true labor? The cervix dilates and becomes effaced in true labor. Bloody show is the first sign of true labor. The membranes rupture at the beginning of true labor. Fetal movements lessen and become weaker in true labor.

The cervix dilates and becomes effaced in true labor. The major difference between true and false labor is that true labor can be confirmed by the presence of dilation and effacement of the cervix. Bloody show may occur before or after true labor begins. The membranes may rupture before or after labor begins. Fetal movements continue unchanged throughout labor.

Which statement indicates that a pregnant client requires further teaching about fetal growth and development? The fetus keeps growing throughout pregnancy. ''The fetus gets nutrients from the amniotic fluid. ''The fetus may be underweight if it's exposed to smoke. ''The fetus gets oxygen from blood coming through the placenta.'

The fetus gets nutrients from the amniotic fluid. The amniotic fluid provides protection, not nutrition; the fetus depends on the placenta, along with the umbilical blood vessels, for nutrients and oxygen. The statements that the fetus keeps growing throughout pregnancy, that it may be underweight if exposed to smoke, and that it gets oxygen from blood in the placenta all indicate that the client understands the teaching.

A 42-year-old client at 39 weeks' gestation has a reactive nonstress test (NST). Which interpretation pertains to this result? Immediate birth is indicated. This is the desired response at this stage of gestation. Further testing is unnecessary with this desired outcome. The result is inconclusive, indicating the need for further evaluation.

This is the desired response at this stage of gestation. An NST indicates that the fetus is healthy because there is an active pattern of fetal heart rate acceleration with movement. The result is positive and desired; immediate birth is not required. Further testing is needed. If the pregnancy continues, another test of fetal well-being will probably be done. The results were positive, not inconclusive.

Which complication is the pregnant client at risk for related to the dilation of renal pelves and ureters? Frequent urination Urinary tract infection Glomerular filtration rate decreases Increased urinary excretion of protein and albumin

Urinary tract infection Dilation of renal pelves and ureters during pregnancy increases the risk of urinary tract infections. Frequent urination is an expected occurrence during pregnancy due to increased bladder sensitivity during early pregnancy and due to bladder compression by the uterus during later pregnancy. By the end of the first trimester the glomerular filtration rate increases by 50% and remains elevated throughout pregnancy. During normal pregnancy there is an increased urinary excretion of protein and albumin, most notably after 20 weeks' gestation.

Which instruction does the nurse give to a client who arrives in the birthing room with the fetal head crowning? Push forcefully. Turn to the left side. Use the pant-breathing pattern. Assume the knee-chest position.

Use the pant-breathing pattern. Panting will slow the process so the nurse can support the head as the baby is born. Pushing will speed the birth, which could result in injury to both mother and fetus. Turning the mother on her left side will have no effect on the progress of the second stage of labor, and it is difficult to accomplish when the fetal head is crowning. Having the mother assume the knee-chest position will have no effect on the progress of the second stage of labor, and it is difficult to accomplish when the fetal head is crowning.

Which task is appropriate for the postpartum nurse to delegate to an unlicensed assistive personnel? Evaluation of a postpartum client's lochia Vital signs on a client 4 hours after delivery Assessment of a postpartum client's episiotomy Assisting the postpartum client to breast-feed for the first time

Vital signs on a client 4 hours after delivery Evaluating the client's lochia, assessing the client's episiotomy, and assiting the client breast-feed for the first time would involve assessment, teaching, or evaluation and should not be delegated. The only task that does not require any of these is taking vital signs 4 hours after delivery.

Which response would the nurse give to a client who asks how far into her pregnancy she can continue to work? ' What activities does your job entail?' 'How do you feel about continuing to work?' 'Most women work throughout their pregnancies.'' Usually women quit work at the start of their third trimester.'

What activities does your job entail?' More information about job activities is needed before the nurse can give a professional response. Although it is important to ascertain the client's feelings about continuing to work, at this time she is seeking information. Although it is true that most women work throughout their pregnancies, more information is needed before the nurse can respond. It is misinformation to state that usually women quit work at the start of the third trimester.

Which nutritional deficiency in pregnant women places the infant at risk for malformations of the central nervous system? Zinc Sodium Potassium Magnesium

Zinc Zinc deficiency in pregnant women is associated with malformations of the central nervous system in infants. Malformations of the central nervous system in infants are not associated with sodium, potassium, or magnesium deficiencies in pregnant women.

In which location will the fetal heart tones be heard if the fetus' position is left occiput anterior (LOA)? (IMAGE) a b c d

d In the most common position, LOA, the fetus's back is on the left side of the mother (position d). Position a is correct when the fetus is in the right sacrum anterior position. Position b is correct when the fetus is in the right occiput posterior position. Position c is correct when the fetus is in the left sacrum anterior position.

Which response would the nurse give to a client who asks what having a fetus in longitudinal lie means in relation to her labor and birth of the baby? 'A vaginal birth is possible.' 'We're anticipating a cesarean delivery.' 'It has no relevance to the labor and birth.' Labor probably will be long, and you might have back pain.'

A vaginal birth is possible.' A longitudinal lie means that the fetus is lying parallel to the woman's spine; therefore vaginal birth is possible. A transverse, not longitudinal, lie might indicate that vaginal birth is unlikely, and cesarean birth is anticipated. The fetal lie will influence the labor and the birth of the fetus. A longitudinal lie does not indicate that the labor will be prolonged; however, if the fetal head is in the posterior occiput position, second-stage labor may be prolonged, accompanied by back pain.

Which recommendation would the nurse make for a pregnant patient experiencing nausea and vomiting? Select all that apply. One, some, or all responses may be correct. Avoid an empty or excessively full stomach. Drink real ginger ale or tea, or use real ginger in another recipe. Try sucking on sour candies or smelling a citrus-scented food or product. Eat crackers or vanilla wafers or drink a small amount of liquid before getting out of bed. Eat small, carbohydrate-rich, low-fat meals throughout the day, such as toast, oatmeal, or noodle soup. Locate the pressure points to reduce nausea located at the middle of the wrist, and press firmly for 3 minutes.

All selection are correct answes: Avoid an empty or excessively full stomach. Drink real ginger ale or tea, or use real ginger in another recipe. Try sucking on sour candies or smelling a citrus-scented food or product. Eat crackers or vanilla wafers or drink a small amount of liquid before getting out of bed. Eat small, carbohydrate-rich, low-fat meals throughout the day, such as toast, oatmeal, or noodle soup. Locate the pressure points to reduce nausea located at the middle of the wrist, and press firmly for 3 minutes. Avoiding complete emptying of the stomach or overfilling the stomach can help reduce bouts of nausea. The oft-repeated home remedy of ginger for nausea is based on fact and worth a try! Sucking on sour candy or even smelling something sour, such as a citrus-scented hand lotion, might help relieve queasiness. Eating a small, bland carbohydrate before rising in the morning cuts back on morning sickness for many expectant mothers. Small, carbohydrate-rich, low-fat meals spread throughout the day would help the mother avoid an empty stomach, which can cause nausea. There are pressure spots on the wrists that might help relieve nausea; alternatively, acupuncture from a trained Eastern medicine specialist might be a consideration.

Which suggestion would the nurse make to a client with morning sickness? 'Eat dry crackers before you get out of bed.' 'Increase your fat intake before bedtime.' 'Drink high-carbohydrate fluids with meals.' 'Eat 2 small meals a day and a snack at noon.'

'Eat dry crackers before you get out of bed.' Nausea and vomiting in the morning occur in almost 50% of all pregnancies. Eating dry crackers before getting out of bed in the morning is a simple remedy that may provide relief. Increasing fat intake does not relieve the nausea. Drinking high- carbohydrate fluids with meals is not helpful; separating fluids from solids at mealtime is more advisable. Eating 2 small meals a day and a snack at noon does not meet the nutritional needs of a pregnant woman, nor will it relieve nausea. Some women find that eating 5 or 6 small meals daily instead of three large ones is helpful.

Which is the appropriate response by the nurse when a 14-year-old emancipated minor says that she does not want her mother present for her prenatal examination? Your mother needs to be present for the examination.' 'What's the problem with your mother being present? ''I'm sure that your mother wants to be with you for support.' 'I will have your mother stay in the waiting area.'

'I will have your mother stay in the waiting area.' In many jurisdictions a minor who is self-supporting and living away from home, providing military service, married, pregnant, or a parent is considered an emancipated minor. The emancipated minor assumes most responsibilities before the age of 18 years. An emancipated minor is entitled to confidentiality in dealings with health care providers. It is appropriate to ask the client's mother to step out of the room. Insisting that the mother stay in the room, asking the minor why she does not want her mother in the room, or suggesting that the mother wants to be in the room are not appropriate responses.

During a routine second-trimester visit to the prenatal clinic a client expresses concern regarding gaining weight and losing her figure. She says to the nurse, 'I'm going on a diet.' Which is the nurse's best response? ' That's fine as long as you include a variety of foods daily.' 'It's a good idea for you to keep your weight down during your pregnancy.' 'If you add 340 calories a day to your regular diet, you won't become overweight.' 'Gain no more than 25 lb (11 kg) so that it'll be easier to lose the weight after the baby is born.'

'If you add 340 calories a day to your regular diet, you won't become overweight.' Weight reduction is not advised during pregnancy; an additional 340 calories a day during the second trimester is recommended. When the client reaches the third trimester, another 120 calories should be added to her diet. A pregnant woman should not diet during pregnancy. Advising the client to eat a variety of foods provides insufficient information. The client should increase her protein and calorie intake during pregnancy. Dieting during pregnancy is harmful; the fetus may be deprived of essential nutrients. The client should not be limited to a specific weight gain. There is no specific recommendation for the amount of weight a pregnant woman should gain. However, 25 to 30 lb (11-16 kg) is the average generally suggested; this figure is based on the recommended caloric intake during pregnancy and the client's prepregnancy weight and metabolic rate.

As the nurse helps a postpartum client change her perineal pad, the client comments, 'I wish you didn't have to look at the pad. It's so embarrassing for me.' Which is the best response by the nurse? 'This seems to be uncomfortable for you; however, I have to estimate the amount of blood loss to identify any potential problems.' 'There can be more blood loss than you might realize. We can determine how much you've lost with a formula.' 'Examining the pad is a common practice that helps us keep you safe. It's a necessary part of the job, and I don't mind.' 'Looking at your pad is a procedure we follow to determine the extent of your bleeding so we can give you the necessary care.'

'This seems to be uncomfortable for you; however, I have to estimate the amount of blood loss to identify any potential problems.' Recognizing the client's discomfort and informing the client of the need to estimate the amount of blood loss acknowledges her feelings and provides an explanation for the intervention. Blood loss can be estimated from the pad count, the degree of saturation, and the time taken for the saturation to occur; an estimate of loss can give the nurse an opportunity to prevent complications caused by hemorrhage. Informing the client that the blood loss can be calculated does not identify the client's feelings; also, this statement may be alarming. Telling the client that examining the pad is a common practice or procedure does not acknowledge the client's feelings; it is a general response that does not educate the client about why this assessment is necessary.

Which statements regarding the involution process are correct? Select all that apply. One, some, or all responses may be correct. Involution begins immediately after expulsion of the placenta. Involution is the self-destruction of excess hypertrophied tissue. Involution progresses rapidly during the next few days after birth. Involution is the return of the uterus to a nonpregnant state after birth. Involution may be caused by retained placental fragments and infections.

Ans: Involution begins immediately after expulsion of the placenta. Involution progresses rapidly during the next few days after birth. Involution is the return of the uterus to a nonpregnant state after birth. The involution process is the return of the uterus to a nonpregnant state after birth; it begins immediately after expulsion of the placenta and contraction of the uterine smooth muscle. This process progresses rapidly during the first few days after birth. Subinvolution is the self-destruction of excess hypertrophied tissue; this process may be caused by retained placental fragments or infection.

For which reason would the nurse encourage a client to void during the first stage of labor? A full bladder is often injured during labor. A full bladder may inhibit the progress of labor. A full bladder jeopardizes the status of the fetus. A full bladder predisposes the client to urinary infection.

Ans: A full bladder may inhibit the progress of labor. A full bladder inhibits the progress of labor by encroaching on the uterine space and impeding the descent of the fetal head. The bladder may become atonic, but is not physically damaged during the course of labor. A full bladder may lead to prolonged labor but generally does not jeopardize fetal status as long as adequate placental perfusion continues. A full bladder during labor does not predispose the client to

The first day of a client's last menstrual period was July 22. Which is the estimated date of birth (EDB)? May 7 April 29 April 22 March 6

Ans: April 29 Her EDB is April 29. Naegeles rule is an indirect, noninvasive method for estimating the date of birth: EDB = last menstrual period + 1 year - 3 months + 7 days. May 7 is beyond the EDB. April 22 and March 6 are both before the EDB.

When assessing a client who gave birth 1 day ago, the nurse finds the fundus is firm at 1 fingerbreadth below the umbilicus and the perineal pad is saturated with lochia rubra. Which is the nurse's next action? Recording these expected findings Obtaining a prescription for an oxytocic medication Asking the client when she last changed the perineal pad Notifying the primary health care provider of excessive bleeding

Ans: Asking the client when she last changed the perineal pad The amount of lochia would be excessive if the pad were saturated in 15 minutes; saturating the pad in 2 hours is considered heavy bleeding. If the pad has not been changed for a longer period, this could account for the large quantity of lochia, so asking the client when she last changed the perineal pad is appropriate. These findings cannot be supported or recorded without additional information. Excessive bleeding cannot be established without more information from the client. Oxytocics are administered for uterine atony; the need for this is not supported by the assessment of a firm fundus.

Which food contains at least 100 mcg of folate per serving? Select all that apply. One, some, or all responses may be correct. Bread Broccoli Cooked pasta Black-eyed peas Ready-to-eat breakfast cereal

Ans: Black-eyed peas Ready-to-eat breakfast cereal Neural tube defects (NTDs), or failures in closure of the neural tube, are more common in infants of women with poor folic acid intake. Proper closure of the neural tube is required for normal formation of the spinal cord, and the neural tube begins to close within the first month of gestation, often before a person realizes she is pregnant. Therefore, all people who are capable of becoming pregnant should take 0.4 mg of folic acid every day, in addition to consuming dietary sources of folate. One-half cup of black-eyed peas contains at least 100 mcg of folate. Ready-to-eat breakfast cereal contains 200 mcg of folate. A slice of bread contains 20 mcg, not 100 mcg of folate. One-half cup of broccoli and a cup of pasta contain 50 mcg, not 100 mcg of folate.

Which instruction would the nurse give to a client in labor who begins to experience dizziness and tingling of her hands? Breathe into her cupped hands. Pant during the next 3 contractions. Hold her breath with the next contraction. Use a fast, deep, or shallow breathing pattern.

Ans: Breathe into her cupped hands. Dizziness and tingling of the hands are signs of respiratory alkalosis, most likely the result of hyperventilating. Breathing into cupped hands or a paper bag promotes the rebreathing of carbon dioxide. Panting during the next 3 contractions could cause the client to hyperventilate more. Holding her breath with the next contraction will not improve the client's respiratory alkalosis. Using a fast, deep, or shallow breathing pattern could cause the client to hyperventilate more.

Which factor distinguishes true labor from false labor? Cervical dilation is evident. Contractions stop when the client walks around. The client's contractions progress only when she is in a side-lying position. Contractions occur immediately after the membranes rupture.

Ans: Cervical dilation is evident. Progressive cervical dilation is the most accurate indication of true labor. With true labor, contractions will increase with activity. Contractions of true labor persist in any position. Contractions may not begin until 24 to 48 hours after the membranes rupture.

One hour after a birth the nurse palpates a client's fundus to determine whether involution is taking place. The fundus is firm, in the midline, and 2 fingerbreadths below the umbilicus. Which would the nurse do next? Encourage the client to void. Notify the health care provider immediately. Massage the uterus and attempt to express clots. Continue periodic assessments and record the findings.

Ans: Continue periodic assessments and record the findings. Immediately after birth the uterus is 2 cm below the umbilicus; during the first several postpartum hours the uterus will rise slowly to just above the level of the umbilicus. These findings are expected, and they should be recorded. Encouraging the client to void is unnecessary; if the bladder is full, the uterus will be higher and pushed to one side. Notifying the health care provider is unnecessary; involution is occurring as expected. Massage is used when the uterus is soft and 'boggy.'

When can a primigravida fetal heartbeat be heard for the first time? A stethoscope at 4 weeks A fetoscope at 10 to 12 weeks Doppler ultrasound after 20 weeks Doppler ultrasound at 10 to 12 weeks

Ans: Doppler ultrasound at 10 to 12 weeks A fetal heartbeat can be obtained at 10 to 12 weeks with electronic Doppler ultrasound. The heartbeat cannot be obtained with a stethoscope, and 4 weeks is too early to hear a fetal heart. A fetoscope cannot pick up the heartbeat until the 17th week. The heart rate can be detected 8 to 10 weeks earlier than 20 weeks.

At which point during a human pregnancy does the embryo become a fetus? During the 8th week of the pregnancy At the end of the 2nd week of pregnancy When the fertilized egg becomes implanted When the products of conception are seen on the ultrasound

Ans: During the 8th week of the pregnancy During the 8th week of pregnancy the organ systems and other structures are developed to the extent that they take the human form; at this time the embryo becomes a fetus and remains so until birth. At the end of the 2nd week of pregnancy, the developing cells are called an embryo. At the time of implantation, the group of developing cells is called a blastocyst. The embryo can be visualized on ultrasound before it becomes a fetus.

Which descriptor would the nurse use when explaining to a client how to time the frequency of contractions? From the end of 1 contraction to the end of the next contraction From the end of 1 contraction to the beginning of the next contraction From the beginning of 1 contraction to the end of the next contraction From the beginning of 1 contraction to the beginning of the next contraction

Ans: From the beginning of 1 contraction to the beginning of the next contraction The frequency of contractions is timed from the beginning of 1 contraction to the beginning of the next; this is the definition of 1 contraction cycle. The beginning, not the end, of a contraction is the starting point for timing the frequency of contractions. The time between the end of 1 contraction and the beginning of the next contraction is the interval between contractions. Timing from the beginning of 1 contraction to the end of the next contraction is too long a time frame and will produce inaccurate information.

Which statement by a breast-feeding mother indicates that the nurse's teaching regarding stimulating the let-down reflex has been successful? I will take a cool shower before each feeding. I will drink a couple of quarts of fat-free milk a day.' I will wear a snug-fitting breast binder day and night. I will apply warm packs and massage my breasts before each feeding.'

Ans: I will apply warm packs and massage my breasts before each feeding.' Applying warm packs and massaging the breasts before each feeding help dilate milk ducts, promote emptying of the breasts, and stimulate further lactation. Taking a cool shower before each feeding will contract the milk ducts and interfere with the let-down reflex. Heavy consumption of milk products is not required to stimulate the production of milk. Breast binders may inhibit lactation by fooling the body into thinking that milk secretion is no longer needed.

According to Naegele rule, which is the expected date of delivery (EDD) of a client whose last menstrual period began on April 15? January 8 January 22 February 8 February 22

Ans: January 22 To determine EDD with the use of Naegele rule, subtract 3 months from the date of the last menstrual period and add 7 days; in this case the EDD is January 22. January 8 is 2 weeks too early according to this formula. February 8 is too late. February 22 would be 1 month past the true EDD.

Between contractions that are 2 to 3 minutes apart and last about 45 seconds the internal fetal monitor shows a fetal heart rate (FHR) of 100 beats/min. Which is the priority nursing action? Notify the health care provider. Resume continuous fetal heart monitoring. Continue to monitor the maternal vital signs.Document the fetal heart rate as an expected response to contractions.

Ans: Notify the health care provider. The expected FHR is 110 to 160 beats/min between contractions. An FHR of 100 beats/min is bradycardia (baseline FHR slower than 110 beats/min) and indicates that the fetus may be compromised, requiring notifying the health care provider and medical intervention. Resuming continuous fetal heart monitoring may be dangerous. The fetus may be compromised, and time should not be spent on monitoring. Continuing to monitor the maternal vital signs is not the priority at this time. Although a fetal heart rate slower than 110 beats/minute should be documented, it is not an expected response.

Which instruction would the nurse include when teaching episiotomy care? Rest with legs elevated at least 2 times a day. Avoid stair climbing for several days after discharge. Perform perineal care after toileting until healing occurs. Continue sitz baths 3 times a day if they provide comfort.

Ans: Perform perineal care after toileting until healing occurs. Performing perineal care after toileting until the episiotomy is healed is critical to the prevention of infection, which is at the core of episiotomy care. Resting is encouraged to promote involution and general recovery from childbirth. Stair climbing may cause some discomfort but is not detrimental to healing. There is no limit to the number of sitz baths per day that the client may take if they provide comfort.

When the fetal monitor is applied to a client's abdomen, it records late decelerations. Which action would the nurse take? Notify the health care provider. Elevate the head of the bed. Reposition the client on her left side. Administer oxygen by way of facemask.

Ans: Reposition the client on her left side. Late decelerations may indicate impaired placental perfusion. Turning the client on her left side relieves pressure on the vena cava and aorta, improving circulation to the placenta. Calling the health care provider is premature. The nurse should notify the practitioner if late decelerations continue after nursing interventions are implemented. Elevating the head of the bed will increase pressure on the vena cava and aorta, further reducing placental perfusion. Oxygen may be administered if placing the client on her left side does not resolve the late decelerations.

Which information would the nurse include in the discharge teaching of a postpartum client? The prenatal Kegel tightening exercises should be continued. A bowel movement may not occur for up to a week after the birth. The episiotomy sutures will be removed at the first postpartum visit. A postpartum checkup should be scheduled as soon as menses returns.

Ans: The prenatal Kegel tightening exercises should be continued. Kegel exercises may be resumed immediately and should be done for the rest of the client's life because they help strengthen muscles needed for urinary continence and may enhance sexual intercourse. Episiotomy sutures do not have to be removed. Bowel movements should spontaneously return in 2 to 3 days after the client gives birth; a delay of bowel movements promotes constipation, perineal discomfort, and trauma. The usual postpartum examination is 6 weeks after birth; the menses may return earlier or later than this and should not be a factor when the client is scheduling a postpartum examination.

Which response would the nurse give to a postpartum client who asks if she can drink a small glass of wine before breast-feeding the first time to help her relax? I think drinking 1 glass of wine won't be a problem. Go ahead.' 'You seem a little tense. Tell me how you feel about breast-feeding.' 'You seem to find it relaxing, but you should try to find another way to relax.' 'I think drinking 1 glass of wine is alright, but you had better check with your health care provider first.'

Ans: You seem a little tense. Tell me how you feel about breast-feeding.' Stating that the client seems tense and initiating a discussion honors the client's feelings and encourages expression of them; there is no reference to alcohol consumption and its relaxing effects. Alcohol ingestion should not be encouraged, because it enters the breast milk. Stating that the client needs to find another way to relax reflects the client's statement but not her underlying feelings. Suggesting that she find another way to relax may make the client defensive and shut off communication. Although alcohol ingestion should not be encouraged because it enters breast milk, the primary health care provider need not be involved because health education is within the role of the nurse.

Which would the nurse recommend to a client who is formula-feeding her infant and complains of discomfort from engorged breasts? Use warm, moist towels as compresses. Express milk from each breast manually. Apply cold packs and a snugly fitting bra. Restrict oral fluid intake to less than a quart a day.

Apply cold packs and a snugly fitting bra. Application of cold relieves discomfort, and a snug bra provides support and aids in pressure atrophy of acini cells so that milk production is suppressed. Warm, moist compresses are suitable for the breast-feeding mother experiencing discomfort from engorgement because it promotes comfort and stimulates milk production. Expressing milk manually is suitable for the breast-feeding mother who is experiencing engorgement, not one who is formula-feeding, because it promotes comfort and stimulates milk production. Restriction of fluids will not prevent engorgement and may cause dehydration.

Which is the optimal nursing intervention to minimize perineal edema after an episiotomy? Applying ice packs Offering warm sitz baths Administering aspirin as needed Elevating the hips on a pillow

Applying ice packs Cold causes vasoconstriction and reduces edema by lessening the accumulation of blood and lymph at the episiotomy site; cold also deadens nerve endings and lessens the pain. Heat therapy alone does not resolve perineal edema. Aspirin is contraindicated in the early postpartum period because of the risk for hemorrhage. Elevating the hips provides minimal perineal relief.

Which is the best advice the nurse can provide to a pregnant woman in her first trimester? Cut down on drugs, alcohol, and cigarettes.' 'Avoid drugs and don't smoke or drink alcohol.' 'Avoid smoking, limit alcohol consumption, and don't take aspirin.' 'Take only prescription drugs, especially in the second and third trimesters.'

Avoid drugs and don't smoke or drink alcohol.' The first trimester is the period when all major embryonic organs are forming; drugs, alcohol, and tobacco may cause major defects. Cutting down on these substances is insufficient; they are teratogens and should be eliminated. Even 1 oz of an alcoholic drink is considered harmful; baby aspirin may be prescribed to some women who are considered at risk for pregnancy-induced hypertension, however, not during the first trimester. Medications, unless absolutely necessary, should be avoided throughout pregnancy; however, the first trimester is most significant.

The nurse is teaching participants in a prenatal class regarding breast- feeding versus formula feeding. A client asks, 'What is the primary advantage of breast-feeding?' Which response is most appropriate? ' Breast-fed infants have fewer infections.' 'Breast-feeding inhibits ovulation in the mother.' 'Breast-fed infants adhere more easily to a feeding schedule.' 'Breast-feeding provides more protein than cow's milk formula does.'

Breast-fed infants have fewer infections.' Maternal antibodies are transferred from the mother in breast milk, providing protection for a longer time than do antibodies transferred to the fetus by way of the placenta. The neonate is protected by the antibodies and thus has fewer infections. The fetus' own antibody system is immature at birth. Women who breast-feed completely (day and night with no supplementary feedings) may avoid ovulation and resumption of the menstrual cycle. Use of formula or solid foods decreases breast-feeding frequency and can lead to ovulation. Ovulation generally occurs before menses, making it difficult to know when the menstrual cycle is resuming. Therefore, breast- feeding is considered one of the least reliable methods of contraception for the new mother. Because of the higher carbohydrate content of breast milk, which is digested rapidly, breast-fed infants wake more frequently than formula-fed infants. Their feeding demands take more time to regulate than do the formula-fed infants'. Breast milk has 1.1 g protein/100 mL; cow's milk has 3.5 g/100 mL. Whole cow's milk is unsuitable for infants.

Which is the most highly sensitive time within the developing embryo for the risk of malformation related to environmental teratogens? Heart at 32 weeks' gestation Cleft lip at 18 weeks' gestation Cleft palate at 8 weeks' gestation Upper limbs at 24 weeks' gestation

Cleft palate at 8 weeks' gestation The most highly sensitive time within the developing human embryo for malformation caused by environmental teratogens is cleft palate at 8 weeks' gestation. The most highly sensitive time for the heart is between 6 and 9 weeks, not at 32 weeks. The most highly sensitive period for the cleft lip is between 5 and 7 weeks, not at 18 weeks. The most highly sensitive time for the upper limbs is between 6 and 9 weeks, not at 24 weeks.

During a nonstress test, the baseline fetal heart rate of 130 to 140 beats per minute rises to 160 twice and 157 once during a 20-minute period. Each of these episodes lasts 20 seconds. Which action would the nurse take? Discontinue the test because the pattern is within the normal range. Encourage the client to drink more fluids to decrease fetal heart rate. Notify the primary health care provider and prepare for an emergency birth. Record this nonreassuring pattern and continue the test for further evaluation.

Discontinue the test because the pattern is within the normal range. The baseline heart rate is within the expected range. The accelerations meet the criteria for an increase of 15 beats that lasts at least 15 seconds during a 20-minute period. This is a reassuring pattern that is indicative of fetal well-being. Drinking more fluids is unnecessary because the fetal heart rate is within the expected range. Preparing for an emergency birth is unnecessary because the test results indicate fetal well-being. The test results meet the standards for a reassuring pattern; further evaluation is unnecessary.

Which instruction would the nurse include when teaching a client about a contraction stress test (CST)? Empty the bladder before the test. Eat nothing for 6 hours after the test. Take the prescribed alprazolam before the test.Be prepared to remain in the hospital for 12 hours after the test.

Empty the bladder before the test. The CST will take 1 to 2 hours, during which time the client is confined to bed. Movement on and off a bedpan during a CST should be avoided, so it is important to empty the bladder before the test. There are no food restrictions before or after this test. Alprazolam may interfere with results of the CST because it will sedate the fetus. If the test is explained in language that the client can comprehend, an anxiolytic should not be necessary. The client may go home 1 hour after the test is completed.

For which reason is an ultrasound done during the first trimester? Estimate fetal age Detect hydrocephalus Rule out congenital defects Approximate fetal linear growth

Estimate fetal age Measurement of the crown-rump length is useful in approximating fetal age in the first trimester. Hydrocephalus cannot be detected during the first trimester. Ultrasonography is used to detect structural defects in the second trimester. It is too early in this pregnancy to determine fetal linear growth.

As a client enters the second stage of labor, fetal monitoring shows early decelerations of the fetal heart rate with a return to the baseline at the end of each contraction. Which is the common cause of this fetal heart rate pattern? Maternal diabetes Fetal cord prolapse Maternal hypotension Fetal head compression

Fetal head compression Early decelerations are expected occurrences as the fetal head passes through the birth canal; the fetal heart rate returns to baseline quickly, indicating fetal well-being. Early decelerations are not related to maternal diabetes. Variable decelerations occur with umbilical cord compression, not prolapse. Maternal hypotension will cause late decelerations because of fetal hypoxia.

A client in active labor is 100% effaced, dilated 3 cm, and at +1 station. Which stage of labor has this client reached? First Latent Second Transitional

First The first stage of labor lasts from the onset of contractions until the cervix is fully dilated at 10 cm. The client is in the early phase of the first stage of labor. There is no latent stage of labor. The second stage of labor lasts from complete dilation to birth. There is no transitional stage of labor; transition is the last phase of the first stage of labor.

When the cervix of a woman in labor is dilated 9 cm, she states that she has the urge to push. Which action would the nurse implement at this time? Having her pant-blow during contractions Placing her legs in stirrups to facilitate pushing Encouraging her to bear down with each contraction Reviewing the pushing techniques taught in childbirth classes

Having her pant-blow during contractions Although there are exceptions, the information given indicates that the best response is inhibiting pushing by having the client use pant-blow breathing. Pushing may cause cervical trauma when the cervix is not completely dilated. It is too early to prepare for the second stage of labor or to have the client bear down with each contraction if the cervix is not fully dilated. At this time the client is completely introverted and will be unreceptive to a review of pushing techniques.

At term a client's hemoglobin level is 10.6 g/dL (106 mmol/L) and her hematocrit is 31%. Which physiological factor accounts for these values? Diuresis Infection Alkalosis Hemodilution

Hemodilution The increase in circulating blood volume during pregnancy is reflected in lower hemoglobin and hematocrit readings (physiological anemia of pregnancy); this represents hemodilution. Diuresis would contribute to hemoconcentration rather than hemodilution. Neither infection nor alkalosis lead to a lower hemoglobin or hematocrit.

Which response would the nurse give to a newly delivered client who asks to take the placenta home with her upon discharge? 'I'll make sure it is kept for you.' 'I'm sorry, but you can't do that.' 'I'll give it to your husband to take home now.' 'I need to check the hospital protocol for our policy on that practice.'

I need to check the hospital protocol for our policy on that practice.' The placenta is a part of the body and contains body fluids. It must first be assessed by the health care provider to be sure that it is not infected and to be sure that all parts of the placenta have been accounted for. The nurse must follow hospital protocol regarding the release of the placenta to the family. All necessary documentation must be signed and the policies must be followed before the release of the placenta to the family.

Which client statement would cause the nurse to stop the health care provider from initiating epidural anesthesia? I'm not exactly sure how an epidural works.' 'I understand that the epidural might or might not take my pain away.' 'I signed the consent form for an epidural at my last clinic appointment.' 'I'm aware that the epidural could cause my contractions to slow down.'

I'm not exactly sure how an epidural works.' A description of the various anesthetic techniques and what they entail is essential to informed consent, even if the woman received information about analgesia and anesthesia earlier in her pregnancy. Nurses play a significant role in the informed consent process by clarifying and describing procedures or by acting as the woman's advocate and asking the primary health care provider for further explanation. There are three essential components of an informed consent. First, the procedure and its advantages and disadvantages must be thoroughly explained. Second, the woman must agree with the plan of labor pain management as explained to her. Third, her consent must be given freely without coercion or manipulation from the health care provider. The patient's statement that she does not understand how the epidural works would cause the nurse to stop the health care provider from initiating epidural anesthesia. Understanding the procedure (that the epidural might or might not take pain away and that it may cause contractions to slow down) would not stop the provider from initiating epidural anesthesia. Signing the consent form would not stop the provider from initiating epidural anesthesia.

While caring for a client during labor, which would the nurse remember about the second stage of labor? It ends at the time of birth. It ends as the placenta is expelled. It begins with the transition phase of labor.It begins with the onset of strong contractions.

It ends at the time of birth. The second stage of labor begins with full cervical dilation and ends with the birth of the infant. The third stage of labor begins after birth, continues until the separation of the placenta from the uterine wall, and ends with the expulsion of the placenta. The transition phase of labor is the last phase of the first stage of labor. The onset of strong contractions occurs during the active phase of the first stage of labor.

Which statement explains the primary purpose of the side-lying position during labor? 'Lying on the side prevents fetal hyperactivity.' 'It makes it less likely that you'll have nausea and vomiting.' 'Lying on the side encourages the presenting part to descend.'' It enhances blood flow to the uterus and makes contractions easier.'

It enhances blood flow to the uterus and makes contractions easier.' In the side-lying position, the gravid uterus does not impede venous return; cardiac output increases, leading to improved uterine perfusion, uterine contractions, and fetal oxygenation. Lying on the side does not affect fetal activity. This position will not ease nausea and vomiting; nausea and vomiting may occur as labor progresses toward the second stage. Walking or squatting will best bring about descent of the presenting part.

The fetus of a client in labor is found to be at +1 station. Where would the nurse locate the presenting part? On the perineum High in the pelvis Just below the ischial spines Slightly above the ischial spines

Just below the ischial spines The term station is used to indicate the location of the presenting part. The level of the tip of the ischial spines is considered 0 station. The position of the bony prominence of the fetal head is described in centimeters minus (above the spines) or plus (below the spines). Just below the ischial spines is a +1 station. On the perineum, referred to as crowning, is designated as +5. High in the pelvis is indicated by the term floating, which means that the presenting part has not yet engaged in the pelvis. A station of -1 indicates that the head is just above the ischial spines.

Which suggestion would the nurse make regarding what a client would wear to prevent back pain as pregnancy progresses? Maternity girdle Support stockings Low-heeled shoes Loose-fitting clothing

Low-heeled shoes Low-heeled supportive shoes help maintain the body's center of gravity over the hips, limiting arching of the back that compensates for the increased weight in the abdominal area. Maternity girdles are no longer recommended. Support stockings may be helpful for a woman with varicose veins or ankle edema; however, wearing them does not prevent back pain. Loose-fitting clothing is more comfortable but has no effect on back pain.

A vaginal examination reveals that a client's cervix is 90% effaced and dilated to 6 cm. The fetus's head is at station 0, and the fetus is in a right occiput anterior position. The contractions are occurring every 3 to 4 minutes, are lasting 60 seconds, and are of moderate intensity. Which description is appropriate to use when reporting on the client's condition? Early first stage of labor Transition stage of labor Beginning second stage of labor Midway through first stage of labor

Midway through first stage of labor The cervix is 90% effaced and dilated 6 cm during the active phase of (or midway through) the first stage of labor. When the cervix is dilated 6 cm, the individual is beyond the early stage of labor. Transition is the last phase of the first stage of labor, which begins when the cervix is dilated 8 cm. The second stage of labor begins when the cervix is fully dilated and 100% effaced.

Which type of lochia would the visiting nurse expect to observe on a client's pad on the third day after a vaginal delivery? Scant alba Scant rubra Moderate rubra Moderate serosa

Moderate rubra The uterus sloughs off the blood, tissue, and mucus of the endometrium postdelivery. This happens in 3 stages that will vary in length and represent the normal healing of the endometrium. Lochia rubra is the first and heaviest stage of lochia. The blood that is expelled during lochia rubra will be bright red and may contain blood clots. The lochia rubra phase typically lasts for about 3 days but may last up to 7 days. Lochia serosa is the second stage of postpartum bleeding and is thinner in consistency and brownish or pink in color. Lochia serosa typically lasts about 2 weeks, although for some women it can last up to 4 to 6 weeks postpartum. Lochia alba is the final stage of lochia; rather than blood, you will see a white or yellowish discharge that is generated during the healing process and the initial reconstruction of the endometrium. Expect this discharge to continue for around 6 weeks after birth, but keep in mind that it may extend beyond that if the second phase of lochia lasted longer than 2 weeks.

Which potential complication is associated with ketonuria that can occur when clients severely restrict their calorie intake during pregnancy? Preterm labor Placenta previa Gestational diabetes Hyperemesis gravidarum

Preterm labor Dietary restriction during pregnancy results in catabolism of fat stores that in turn augments the production of ketones, and ketonuria is associated with preterm labor. Ketonuria is not associated with placenta previa, gestational diabetes, or hyperemesis gravidarum.

Which effect does the nurse expect after an amniotomy is performed on a client in active labor? Diminished vaginal bleeding Less discomfort with contractions Progressive dilation and effacement Increased maternal and fetal heart rates

Progressive dilation and effacement Amniotomy permits more effective pressure of the fetal head on the cervix, enhancing dilation and effacement. Vaginal bleeding may increase because of the progression of labor. Discomfort may increase because contractions usually become more intense after amniotomy. Amniotomy should not affect maternal and fetal heart rates.

Which physiological changes would the nurse anticipate after an amniotomy is performed? Diminished bloody show Increased and more variable fetal heart rate Less discomfort with contractions Progressive dilation and effacement

Progressive dilation and effacement Artificial rupture of the membranes (amniotomy) allows more effective exertion of pressure of the fetal head on the cervix, enhancing dilation and effacement. Vaginal bleeding (bloody show) may increase because of the progression of labor. Amniotomy does not directly affect the fetal heart rate. Discomfort may become greater because contractions usually increase in intensity and frequency after the membranes are artificially ruptured.

A client who is 38 weeks' pregnant has a nonstress test (NST). The resulting fetal monitor strip is shown. Which interpretation would the nurse assign to this finding? Negative because of the lack of contractions Nonreassuring; fetal heart rate lacks variability Reassuring; fetal heart rate accelerates with movement Positive; demonstrates decelerations with fetal movement

Reassuring; fetal heart rate accelerates with movement The NST is used to compare fetal heart accelerations with fetal movement. This strip shows adequate fetal heart rate variability, and the heart rate accelerates with fetal movement, which is reassuring. Contractions are not a component of an NST. The strip shows adequate fetal heart rate variability, so it is not nonreassuring. A positive NST indicates decelerations of the fetal heart rate with uterine contractions.

Which action would the nurse take when a 15-beat-per-minute acceleration of the fetal heart rate above the baseline occurs during a contraction? Call the practitioner to prepare for an imminent birth. Turn the mother on her left side to increase venous return. Record the fetal response to contractions and continue to monitor the heart rate. Document the fetal heart rate abnormality and monitor the fetal heart rate continuously.

Record the fetal response to contractions and continue to monitor the heart rate. Periodic accelerations are the most reassuring of fetal heart rate indicators, regardless of the cause. The fetal response is recorded and monitoring continues unchanged. This increase in the fetal heart rate does not require intervention by the practitioner at this time. Turning the mother on her left side to increase venous return is done when a fetal heart rate deceleration occurs. This acceleration is not a fetal heart rate abnormality and does not require a specific frequency of monitoring.

Which of these presentations would indicate that the nurse should direct a primipara to call a health care provider? Bloody show or back pressure occurring with no contractions Irregular contractions coming 10 minutes apart Rupture of membranes or contractions 5 minutes apart Contractions 12 minutes apart and lasting about 30 seconds

Rupture of membranes or contractions 5 minutes apart When the membranes rupture, the potential for infection is increased, and when the contractions are 5 to 8 minutes apart, they are usually of sufficient force to warrant professional supervision. Bloody show and back pressure may be early signs of labor or signs of posterior fetal position; however, it is too early to notify the health care provider. Irregular contractions coming 10 minutes apart and contractions 12 minutes apart and lasting about 30 seconds indicate that it is too soon in the labor process to call the health care provider; the client should remain with her family and keep moving around at home.

Which routine screening would the nurse perform specifically for pregnant adolescents? Sexual assault Alcohol abuse Substance use Occupational risks

Sexual assault Routine screening for sexual assault and abuse is recommended for pregnant adolescents because pregnancy in minor adolescent girls can be the result of sexual assault and abuse. Screening for alcohol abuse, substance use, and occupational risks should be performed for all pregnant women, not specifically for pregnant adolescents.

The electronic fetal monitor on a client receiving an infusion of oxytocin (Pitocin) displays contractions every 2 minutes and lasting 95 seconds. Which is the appropriate nursing action? Stop the oxytocin (Pitocin) infusion. Administer oxygen at 8 to 10 L/min. Increase the main line fluid delivery rate to 150 mL/hr. Prepare the client for insertion of an intrauterine pressure catheter.

Stop the oxytocin (Pitocin) infusion. The contraction pattern indicates hyperstimulation of the uterus. Stopping the oxytocin (Pitocin) infusion permits relaxation of the uterus and perfusion of the placenta. Oxygen cannot reach the placenta until the uterus is relaxed, so administering oxygen will not help. Increasing the rate of delivery of the main line fluid does not affect hyperstimulation of the uterus. Insertion of an intrauterine pressure catheter will only provide measurement of the internal uterine pressure and will not affect uterine contractions.

When palpating a client's fundus on the second postpartum day, the nurse determines that it is above the umbilicus and displaced to the right. Which conclusion is supported by this finding? There is a slow rate of involution. There are retained placental fragments. The bladder has become distended. The uterine ligaments are overstretched.

The bladder has become distended. A distended bladder will displace the fundus upward and laterally to the right. A slow rate of involution is manifested by slow contractions and uterine descent into the pelvis. If retained placental fragments were present, the uterus would be boggy in addition to being displaced and vaginal bleeding would be heavy. From this assessment the nurse cannot make a judgment regarding overstretched uterine ligaments.

Which cervical changes are observed during pregnancy? Select all that apply. One, some, or all responses may be correct. The cervical tip becomes soft. The fragility of cervical tissues decreases. The volume of cervical muscle increases. The external cervical os appears as a jagged slit. The elasticity of cervical collagen-rich connective tissue increases.

The cervical tip becomes soft. The volume of cervical muscle increases. The elasticity of cervical collagen-rich connective tissue increases. By the beginning of the sixth week of pregnancy, the cervical tip softens. During pregnancy, the cervical muscle and its collagen-rich connective tissues increase in volume and become loose and highly elastic. Cervical tissue fragility also increases. The external cervical os appears as a jagged slit postpartum; however, it does not during pregnancy.

A client in active labor is admitted to the birthing room. A vaginal examination reveals that her cervix is dilated 6 to 7 cm. In light of this finding, which would the nurse expect? The client may experience nausea and vomiting. The client's bloody show will become more profuse. The client will experience uncontrollable shaking of her legs. The client's contractions will become longer and more frequent.

The client's contractions will become longer and more frequent. The nurse should expect the client's contractions to become longer and more frequent as labor progresses through the active portion of the first stage of labor. Nausea and vomiting occur in the transition phase of the first stage of labor. More profuse bloody show and uncontrollable shaking of the legs occur in the transition phase of the first stage of labor.

A pregnant client asks how smoking will affect her baby. Which information about cigarette smoking will influence the nurse's response? It relieves maternal tension, and the fetus responds accordingly to the reduction in stress. The resulting vasoconstriction affects both fetal and maternal blood vessels. Substances contained in smoke permeate through the placenta and compromise the fetus's well-being. Effects are limited because fetal circulation and maternal circulation are separated by the placental barrier.

The resulting vasoconstriction affects both fetal and maternal blood vessels. Cigarette smoking or continued exposure to secondary smoke causes both maternal and fetal vasoconstriction, resulting in fetal growth retardation and increased fetal and infant mortality. There is no clinical evidence that smoking relieves tension or that the fetus is more relaxed. Smoking causes vasoconstriction; permeability of the placenta to smoke is irrelevant. Although the fetal and maternal circulations are separate, vasoconstriction occurs in both mother and fetus.

A client in active labor is considering combined spinal-epidural analgesia and states that she is concerned about her ability to walk after receiving this type of analgesia. Which is the most accurate response by the nurse? ' This analgesia gives you pain relief without compromising your ability to ambulate.' 'The analgesia will require you to remain in bed, but you'll be able to move from side to side.' 'You may experience slight weakness, but someone will be at your side when you're ambulating.' 'Someone will help you ambulate every couple of hours before you're given another dose of the analgesia.'

This analgesia gives you pain relief without compromising your ability to ambulate.' Because the spinal nerve receptors are sensitive to opioids, small quantities are needed to produce analgesia; therefore, the client's ability to ambulate without assistance is not jeopardized. Bed rest is not required. The analgesia does not cause weakness. The client's ability to ambulate without assistance is not jeopardized. The analgesia lasts for more than 2 hours; there is no need to ambulate before a dose is given.

For which condition are pregnant women at a five- to sixfold increased risk? Bradycardia Hypertension Thromboembolic disease Decreased cardiac output

Thromboembolic disease Pregnancy is considered a hypercoagulable state, which places pregnant women at a five- to sixfold increased risk of thromboembolic disease. Heart rate increases 10 to 15 beats/minute during pregnancy. Systolic blood pressure increases only slightly or decreases during pregnancy, and diastolic blood pressure slightly decreases by midpregnancy and returns to prepregnancy levels by the end of pregnancy. Cardiac output increases, not decreases 30% to 50% during pregnancy.

In the 8th month of pregnancy a client tells the nurse that she is experiencing dyspareunia. Which information would be most helpful for the nurse to teach the client? ' Avoid intercourse.' 'Try alternative positions.' 'Consult a therapist for sex counseling.' 'Douche to lubricate the vaginal mucosa.'

Try alternative positions.' Pain caused by deep penetration by the male partner is common in late pregnancy and can be reduced with the use of alternative positions such as rear entry. Avoiding intercourse or consulting a therapist should not be suggested until other alternatives have been tried. Douching is not recommended and does not lubricate the vagina; a water-soluble lubricant is more effective.

Which recommendation would the nurse make to a pregnant client who sits almost continuously during her working hours? 'Try to walk around every few hours during the workday.' 'Ask for time in the morning and afternoon to elevate your legs.' 'Tell your boss that you won't be able to work beyond the second trimester.' 'Ask for time in the morning and afternoon so you can go get something to eat.'

Try to walk around every few hours during the workday.' Maintaining the sitting position for prolonged periods may constrict the vessels of the legs, particularly those in the popliteal spaces, and may diminish venous return. Walking causes the leg muscles to contract and applies gentle pressure to the veins, thereby promoting venous return. Walking around several times each morning and afternoon will improve circulation; the legs may be elevated while the client is sitting at her desk. If the client is feeling well, there are no contraindications to working throughout her pregnancy. Adequate nourishment can be obtained during mealtimes; the client does not require extra nutrition breaks.

Which result after 20 minutes of a nonstress test is suggestive of fetal reactivity? Absence of long-term variability Above-average fetal baseline heart rate of 160 beats/min No late decelerations associated with contractions Two accelerations of 15 beats/min lasting 15 seconds

Two accelerations of 15 beats/min lasting 15 seconds According to the American Congress of Obstetricians and Gynecologists, fetal reactivity is a fetal tracing 15 beats' acceleration above baseline lasting 15 seconds or more, normal baseline rate, and long-term variability amplitude of 10 or more beats/min. An absence of long-term variability is an ominous sign that must be addressed. An above-average baseline heart rate is acceptable up to 160 beats/min. An increasing baseline heart rate is a sign of maternal infection. Contractions are not expected with a nonstress test; early, late, or variable fetal heart rate decelerations are associated with uterine contractions.

When the nurse asks participants in a prenatal class to demonstrate effleurage, which behavior would be observed? Panting and blowing Slow, deep breathing Rocking back and forth gently on their knees Using the fingertips to gently massage their abdomens

Using the fingertips to gently massage their abdomens Effleurage is a gentle massage of the abdomen that is effective during the first stage of labor because it distracts the client from the discomfort of the contractions. Panting and blowing is a breathing technique to avoid pushing during labor. Taking slow, deep breaths is also a technique of breathing. Rocking gently on the knees, known as the pelvic rock, is used during pregnancy to relieve backache.

Which weight change in the obese prenatal client during the 6th month of pregnancy would indicate to the nurse that the client is successfully managing her weight gain? Weight loss of 1 lb (0.45 kg) Weight gain of 2 lb (0.91 kg) Weight gain of 5 lb (2.26 kg) The client's statement that she lost weight last week

Weight gain of 2 lb (0.91 kg) Although obese, the client must gain some weight to meet the fetus's nutritional needs, and a 2-lb (0.91-kg) weight gain is appropriate. Weight gain recommendations in pregnancy are based on the client's prepregnancy body mass index (BMI), and for the obese patient, the recommended total weight gain is 11 to 20 lb (4.9-9.7 kg) Weight loss is contraindicated during pregnancy because it may interfere with fetal growth and development. A 5-lb (2.26-kg) weight gain in 1 month would be excessive. The client's statement that she lost weight last week does not constitute objective data.

A client at 35 weeks' gestation asks the nurse why her breathing has become more difficult. How would the nurse respond? 'Your lower rib cage is more restricted. ''Your diaphragm has been displaced upward. ''Your lungs have increased in size since you got pregnant.' The height of your rib cage has increased since you got pregnant.'

Your diaphragm has been displaced upward. The pressure of the enlarging fetus causes upward displacement of the diaphragm, which results in thoracic breathing; this limits the descent of the diaphragm on inspiration. The lower rib cage expands; it does not become restricted. There is no change in the size of the lungs during pregnancy. The thoracic cage enlarges; it does not rise.

A client whose weight was average for her height before becoming pregnant expresses concern about her 15-lb (6.8-kg) weight gain after only 23 weeks of pregnancy. Which is an appropriate response? ' Actually, you have not gained enough weight. Try to increase your daily intake of calories.' 'Your weight is not a concern, but I can refer you to the dietitian for a diet review if you like.' 'You've gained too much weight for 23 weeks' gestation. You need to avoid salty and fatty foods.' 'Your weight is as expected for someone at 23 weeks' gestation, so continue with your current diet.'

Your weight is as expected for someone at 23 weeks' gestation, so continue with your current diet.' The recommended average weight gain is 2.2 to 5.5 lb (1-2.5 kg) during the first 12 weeks, then approximately 1 lb (0.45 kg) per week until birth; 14 to 16 lb (6.4-7.3 kg) is an appropriate weight gain at 23 weeks' gestation. Stating that the client has not gained enough weight is inaccurate information. Stating that the weight is not a concern dismisses the client's concern; also, the nurse is abdicating the responsibility for teaching by the referral to the dietitian. Stating that the client has gained too much weight for 23 weeks' gestation is inaccurate information that may produce anxiety.


Set pelajaran terkait

bariatric surgery nclex questions NUR 212

View Set

Wk 1 OB Prep U Ch 11 Maternal Adaptation During Pregnancy

View Set

Chapter 8 Pharmacology Questions

View Set

Anxiety Disorders and Obesessive-Compulsive and Related Disorders

View Set